You are on page 1of 43

PSYCHIATRY

General Principles 5% - 10%


Organ Systems 90% - 95%
- Mental Disorders 75% - 85%
o Promoting Health and Health Maintenance 1% - 5%
o Understanding Mechanisms of Disease 10% - 15%
o Establishing a Diagnosis 55% - 65%
Mental disorders usually 1st diagnosed in infancy, childhood, or adolescence (5-10%)
Substance-related disorders (5-10%)
Schizophrenia and other psychotic disorders (5-10%)
Mood disorders (5-10%)
Anxiety disorders (5-10%)
Somatoform disorders (1-5%)
Other disorders/conditions (5-10%)
o Applying Principles of Management 20-25%
- Diseases of the Nervous System and Special Senses 10%-15%

Review Session:
- LOTS of questions of substance abuse
- Depression, mood, anxiety
o Suicide risk factors
When the pt can name something that has changed you are in
a better postion than when they cant
Acute change in job, worthlessness hopelessness, substance
abuse, suicide ideation, attempt, plan, older Caucasian males,
poor family support, prior history of suicide, family members
with suicide history
- Body dysmorphic disorder is a 1 question topic
- Child questions about 25% are child/adolescent as patient. (does not mean
child disorders could be MDD, anorexia)
- No theorists
- There will be psychotherapy stuff there wont be freud or theories, no
defense mechanisms
- Everything will be tested on DSM diagnoses.
- Schizophrenia progression brief, schizophreniform, schizophrenia
Neuro:
- dementias
- MR and childhood presentations of MR, prognosis
One question disorders:
- Somatoform - somatization disorder, conversion d/o
o Conversion with OC people loss of fxn tx with appropriate therapy
(PT for poor movement of arm, speech therapist for poor talking, etc)
o Hypochondriasis with feelers reassurance (anxiety based and can
treat with SSRIs in real life.
o Somatization o Body Dysmorphic disorders
- Eating Disorders - Bulimia can be tx with ssri, anorexia tx behavioral
- Sleep disorders
- Pain disorder

dissociative disorders

Key Points:
- Adjustment disorder vs. MDD vs. GAD vs. Acute Stress disorder vs. PTSD
- Medications, ESPECIALLY side effects
- Medical Disorders with psychiatric manifestations
o Depression and pancreatic cancer, addisons disease
o Pheochromocytoma or carcinoid looking like panic disorder
- Depression vs. bereavement vs. somatization disorder
- Physical features of common MR presentations (down syndrome, fragile X)
- Look for delirium and psychosis
- Medication Side effects
o At receptor level
- Neurotransmitters
o Affected in psychiatric disease
Anxiety increased NE, decreased GABA and serotonin
Schizophrenia elevated serotonin, elevated NE, decreased GABA, excess dopamine in
prefrontal cortex = negative sx. Dopamine in mesolimbic pathway = positive symptoms
Blocking dopamine in Tubuloinfundibular hyperprolactinemia; Nigrostriatal
EPS
Depression decreased serotonin, NE, dopamine; high cortisol, abnormal thyroid axis
Panic Disorder increased NE, decreased GABA, serotonin
Alzheimers dementia decreased Ach due to loss of cholinergic neurons in basal
nucleus of meynert in midbrain and decreased NE due to loss of noradrenergic neurons in
locus ceruleus of brainstem.
Parkinson Disease loss of cells in substantia nigre of basal ganglia which leads to
decreased dopamine and loss of dopaminergic tracts
Tourettes/Tics impaired regularion of dopamine in the caudate nucleus and possible
upregularion of endogenous opiates and the noradrenergic system
o Site of NT release
Locus ceruleus NE
Raphe nucleus serotonin
- Intoxication and Withdrawal Section from first aid

PSYCHOTIC DISORDERS
Schizophrenia
To make the diagnosis of schizophrenia, a pt must have symptoms of the disease for at least _____.
- 6 months
Name the 5 subtypes of schizophrenia.
- Paranoid
- Disorganzied
- Catatonic
- Undifferentiated
- residual
What are 5 characteristics of catatonic schizophrenia?
- Motor immobility
- excessive purposeless motor activity
- extreme negativism or mutism
- peculiar voluntary movements or posturing
- echolalia/echopraxia
What are the "5 A's" of schizophrenia, which describe the negative symptoms of the disease?
- Anhedonia
- Affect (flat)
- Alogia (poverty of speech)

- Avolition (apathy)
- Attention (poor)
2 or more of what 5 symptoms must be present for at least 1 month to diagnose schizophrenia?
- Delusions
- Hallucinations
- disorganized speech
- grossly disorganized or catatonic behavior
- negative symptoms
Differentiate between brief psychotic disorder, schizophreniform disorder, and schizophrenia based on duration of
symptoms.
- Brief psychotic disorder: < 1 month
- schizophreniform disorder: 1 - 6 months
- schizophrenia: > 6 months
What are 'ideas of reference'?
- Belief that cues in the external environment are uniquely related to the individual
What is the downward drift hypothesis regarding schizophrenics?
- People suffering from schizophrenia are unable to function well in society and hence enter lower
socioeconomic groups
What are 2 theorized dopamine pathways affected in schizophrenia?
- Prefrontal cortical: inadequate dopaminergic activity responsible for negative symptoms
- mesolimbic: excessive dopaminergic activity responsible for positive symptoms
What are 4 components of the mesolimbic pathway responsible for positive schizophrenia sx?
- Nucleus accumbens
- Fornix
- Amygdala
- hippocampus
The negative sx of schizophrenia are thought to occur due to dopamine action in the _____ pathway.
- Mesocortical
Describe the relative levels of serotonin, norepinephrine, GABA, and glutamate receptors in schizophrenic pts.
- Elevated: serotonin, norepinephrine
- decreased: GABA, glutamate receptors
Name 4 first-generation antipsychotic medications used to treat schizophrenia.
- Chlorpromazine
- Thioridazine
- Trifluoperazine
- haloperidol
First-generation antipsychotic medications are all _____ antagonists.
- Dopamine (mostly D2)
What are 4 second-generation antipsychotic medications used to treat schizophrenia?
- Risperidone; clozapine; olanzapine; quetiapine; aripiprazole; ziprosidone
Second-generation antipsychotic medications antagonize _____ receptors as well as dopamine receptors.
- Serotonin
What are the hallmark side effects for first and second-generation antipsychotics?
- First-generation: EPS; second-generation: metabolic syndrome
What 2 antiparkinsonian medications can be used to treat EPS?
- Benztropine
- diphenhydramine
Blockage of what dopamine pathway by neuroleptic medication results in extrapyramidal side effects?
- Nigrostriatal
In what 3 situations is only one psychotic sx necessary for a diagnosis of schizophrenia?
- Bizarre delusions
- auditory hallucinations commenting on the patient
- two or more voices speaking to each other
Schizoaffective Disorder
What criteria is used to distinguish schizoaffective disorder from mood disorder w/ psychotic features?

Delusions or hallucinations for 2 weeks in the absence of mood disorder symptoms

Schizophreniform Disorder
Brief Reactive Psychosis
MOOD DISORDERS
Bipolar Disorder
Mania:
Describe the DIG FAST mnemonic for symptoms of mania.
- Distractibility
- Insomnia/Impulsive behavior
- Grandiosity
- Flight of ideas/racing thoughts
- Activity/agitation
- Speech (pressured)
- Thoughtlessness
What criterion is required for diagnosis of a manic episode?
- Presence of an abnormally elevated, expansive, or irritable mood and 3 of the above symptoms lasting at
least 1 week
What 3 drugs are approved for the acute treatment of mania?
- Lithium
- Valproate
- carbamazepine
What drug is used as maintenance treatment for bipolar disorder?
- Lamotrigine
All second-generation anti-psychotic medications are approved to treat acute mania except _____.
- Clozapine
What is the rapid-cycling specifier?
- At least 4 major depressive, manic, hypomanic, or mixed episodes within the past 12 months
What time period is required for diagnosis of mania vs. hypomania?
- Mania: 7 days
- hypomania: 4 days
What is the definition of rapid cycling?
- The occurrence of 4 or more mood episodes in 1 year (major depressive, manic, mixed, etc.)
(T/F) Bipolar I has the highest genetic link of all major psychiatric disorders.
- True
What is the best treatment for a manic woman in pregnancy?
- ECT
What 2 anticonvulsants are especially useful for rapid cycling bipolar disorder?
- Carbamazepine; valproic acid
Bipolar depression
Mood stabilizer.
Cyclothymia
Depressive Disorders
Bereavement/Grief
What time period signifies transition from normal to complicated/prolonged grief?
- 6 months
(T/F) Normal bereavement may include gross disorganization or suicidality.
- False
PostPartum Depression
Describe the postpartum onset specifier.
- Depressive, manic, or mixed episode within the first 4 weeks postpartum
(T/F) Postpartum depression usually resolves w/o medication.

- True
What time of onset is required for the postpartum specifier for depression?
- Onset within 4 weeks of delivery
Seasonal Affective Disorder
What is the triad for seasonal affective disorder?
- Irritability
- carbohydrate craving
- hypersomnia
Major Depressive Disorder
What is the response rate of patients w/ major depression to antidepressants?
- 70% will respond; 30% of this response is a placebo effect
- take 4-8 weeks to work (up to 3 months in elderly patients)
What 2 characteristics define major depression?
- Depressed mood; loss of interest or pleasure
Most depressive episodes clear spontaneously within _____.
- 6 months
Name 4 unique subcategories of depressive disorders.
- Melancholic anhedonia, early AM awakenings, PMR, excessive guilt, anorexia
- Atypical hypersomnia, hyperphagia, reactive mood, leaden paralysis, hypersensitive to interpersonal
rejection
- Catatonic catalepsy (immobility), purposeless motor activity, extreme negativism or mutism, bizarre
postures, echolalia; tx with antidepressants and antipsychotics concurrently
psychotic - delusions/hallucinations
Name the 2 characteristics required for the diagnosis of melancholia?
- Pervasive loss of interest or pleasure; inability to respond to pleasurable stimuli
What are 2 unique descriptive features of melancholia?
- Diurnal variation; terminal insomnia
What are 5 characteristics of atypical depression?
- Hypersomnia
- Hyperphagia
- reactive mood
- leaden paralysis
- hypersensitivity to interpersonal rejection
What category of antidepressant medication has been shown to be particularly effective in treating atypical
depression?
- Monoamine oxidase inhibitors (MAOIs)
Name 3 sleep changes a/w depression.
- Decreased delta sleep; decreased REM latency; increased REM density
Describe the progression of anti-depressant drug trials.
- Drug trials should last from 4-8 weeks; if the pt fails to respond within 4 weeks of treatment, the dosage
should be increased or the pt should be switched to another drug, preferably from another class
Augmentation of anti-depressant drugs w/ _____ is the best-researched option.
- Lithium carbonate
What second-generation anti-psychotic medication is often used as anti-depressant augmentation?
- Aripiprazole
_____ has the highest rate of suicide of any psychiatric disorder.
- MDD
What scale is used as the standard measure of depression severity in research to assess the effectiveness of
therapies?
- Hamilton Rating Score
What cancer has a high association w/ depression?
- Pancreatic cancer
Use of MAOIs is a/w what 2 adverse conditions?
- Hypertensive crisis

- serotonin syndrome
(T/F) Treatment of MDD w/ psychotic features may be treated w/ an anti-depressant alone.
- False; treatment should include an anti-depressant as well as either an antipsychotic medication or ECT
What is the protocol for medication changes in a child w/ major depression w/ psychotic features who responds to
the combination of an antidepressant and an antipsychotic?
- Atypical antipsychotic should be continued for 3 months and then tapered off; the antidepressant should be
continued for 6 - 9 months and then tapered over 2 - 3 months at 33% per month
Describe results of a PET scan in a patient w/ depression.
- Reduced metabolic activity and blood flow in both frontal lobes
Use of tetracycline, nifedipine, and verapamil is a/w _____.
- Depression
Treatment of major depression in older adults is most successful with a combination of antidepressant
medications plus interpersonal supportive psychotherapy. Interpersonal psychotherapy is suitable for individuals
having experienced recent interpersonal conflicts or difficult role transitions.
Dysthymic Disorder
How does the diagnosis of dysthymic disorder differ between children and adults?
- Dysthymic disorder can be diagnosed in children if they are symptomatic over a 1 year time period (instead
of the 2 years required for adults)
Electroconvulsive Therapy
Name 4 indications for electroconvulsive therapy (ECT).
- Very severe depression
- high potential for suicide
- cardiovascular disease
- pregnancy
- unresponsive to pharmacotherapy
Describe 2 characteristics of MDD in which ECT treatment would be indicated.
- MDD w/ psychotic features
- MDD where rapid response is required
The effectiveness of ECT is based on _____, not _____.
- Length of postictal suppression; seizure duration
Method:
- premedication with atropine generalized anesthesia muscle relaxant generalized seizure by
current across brain (bilateral/unilateral) seizure < 1 minute.
- 8 tx over 2-3 week period.
- Retrograde amnesia common SE but disappears within 6 months.
NO true contraindications increased intracranial pressure is the most detrimental/associated. Recent MI is not an
absolute contraindication.
ANXIETY DISORDERS
Panic Disorder with and without agoraphobia
What is the hallmark of panic disorder?
- Unexpected panic attacks not provoked by any particular stimulus
The treatment regimen for panic disorder should contain what 2 items?
- Antidepressant; CBT
What other disorder is commonly seen in pts w/ panic disorder?
- MDD
Name 5 medical conditions a/w panic attacks.
- Mitral valve prolapse
- Asthma
- pulmonary embolus
- angina
- anaphylaxis

Name 3 characteristics a/w panic disorder.


- Persistent concern about having additional attacks
- worry about the implications of the attack
- significant change in behavior related to the attacks
What specifier is required w/ the diagnosis of panic disorder?
- W/ or w/o agoraphobia
What 2 SSRIs are commonly used to treat panic disorder?
- Paroxetine
- sertraline
Nothing treats a panic attack like a BZD but dont start person who gets addicted to benzos or not?
Specific and Social Phobias
What qualifier is required for diagnosis of phobias in pts under age 18?
- Duration > 6 months
What SSRI is FDA approved for the treatment of social phobia?
- Paroxetine
Social phobia gradual desensitization; propranolol for physical manifestations; does not take care of actual
anxiety
School phobia in children - flooding
Four most common mental disorders:
1. phobias
2. substance induced disorders
3. major depression
4. OCD
Obsessive Compulsive Disorders
What TCA has been shown to be effective in treating OCD?
- Clomipramine
Differentiate between an obsession and a compulsion.
- Obsession: recurrent and intrusive thought, feeling, or idea that is egodystonic
- compulsion: conscious repetitive behavior linked to an obsession that, when performed, functions to relieve
anxiety caused by the obsession
The rate of OCD is higher in pts w/ first-degree relatives who have _____.
- Tourette syndrome
Post Traumatic Stress Disorder
What 2 classes of medications should be used to treat PTSD?
- SSRIs; alpha-2 agonists (e.g. prazosin, clonidine)
Nightmares, flashbacks, startle responses
Treatment is TALKING - psychoanalytic psychotherapy pays particular attention to the role of trauma and
sexual experiences in shaping both unconscious content and unconscious processing.
Generalized Anxiety Disorder
What atypical antidepressant is used for treatment of generalized anxiety disorder?
- Buspirone
Describe the indication and mechanism of buspirone.
- Non-sedating non-benzodiazepine anxiolytic approved by the FDA for the treatment of anxiety disorders
- acts as an agonist on 5HT-1A receptors
What 4 medications/medication classes are used in the treatment of GAD?
- SSRIs; venlafaxine; buspirone; benzodiazepines
(T/F) Buspirone is not as effective in pts already exposed to benzodiazepines.
- true
Describe the anxiolytic action of buspirone.
- 5HT-1A receptor partial agonist
What is the advantage of buspirone over benzodiazepines?

Does not potentiate the CNS depression of alcohol (useful in alcoholics) and has a low potential for
abuse/addiction
What medication is useful for patients who want a quick-acting, short-term medication but cannot tolerate
benzodiazepines for various reasons?
- Hydroxyzine (antihistamine)
Treatment:
- Psychotherapy
CHILD PSYCHIATRY
Mental Retardation/Intelligence/Cognition
What is the most commonly used IQ test for pts 16 - 75 years of age?
- Wechsler Adult Intelligence Scale (WAIS)
What IQ test is used in pts 2 - 18 years of age?
- Stanford-Binet Test
What 2 intelligence tests are used for children and adolescents?
- Ages 2-12: Kaufman Assessment Battery for Children (K-ABC)
- ages 6-16: Wechsler Intelligence Scale for Children-Revised (WISC-R)
What 3 characteristics are required for diagnosis of mental retardation?
- IQ < 70
- deficits in adaptive skills appropriate for the age group
- onset before the age of 18
Name and describe the IQ levels which correspond to the 4 categories of mental retardation.
- Mild: 70-55
- moderate: 55-40
- severe: 40-25
- profound: < 25
What is the most common inherited form of mental retardation? What is its cause?
- Fragile X syndrome, caused by a defect in the FMR-1 gene
What is the most common learning disorder?
- Reading disorder, affecting up to 10% of school-age children
Name 4 pervasive developmental disorders (PDDs).
- Autistic disorder
- Asperger disorder
- Rett disorder
- childhood disintegrative disorder
Noonan Syndrome - An 8-year-old girl is brought to your office after her mother noticed that she was not growing
well. The mother states that the girl gets poor grades at school, catches cold and cough infections very often, and
bruises very easily. You note that the girl has a triangular face, hypertelorism, deeply grooved philtrum, and downslanting eyes. You also note the presence of webbed neck and low-set ears. The girl has scoliosis and an IQ of 65.
You then order a complete work up, including CBC, coagulation profile, and cardiac evaluation. Echocardiography
detects pulmonary valve stenosis. Suspecting a congenital disorder, you order karyotyping and mutation analysis.
The diagnosis in this girl is Noonan syndrome (NS). PTPN11 gene mutations are seen in 50% of individuals with
NS. SOS1 and KRAS mutations are other causes, although absence of a mutation does not rule out NS.NS is a
sporadic or autosomal dominant congenital disorder with typical phenotypic features, which may not be visible to
the casual onlooker. The most common facial features include hypertelorism and low-set and backward rotated
ears, along with a thick helix. The philtrum is deeply grooved in more than 90% of cases. Congenital cardiac
defects, bleeding disorders, mental retardation, webbed neck, and short stature are other features. Pulmonary valve
stenosis is the most common cardiac defect and is seen in 50% of those affected. Other cardiac findings include
atrial and/or ventricular septal defects and cardiomyopathies.
Mowat-Wilson syndrome refers to an autosomal dominant condition resulting from mutation or deletion of ZEB2
gene on chromosome 22. Characteristic features include: narrow chin; deep, widely set eyes; open mouth;
uplifted ear lobes; wide nasal bridges; and a shortened philtrum. Mental retardation, delayed growth and motor
development, heart disease, and Hirschsprung disease are also seen.

Mutation in the fragile mental retardation 1 (FMR1) gene causes fragile X syndrome (FXS). FXS is the most
common cause of inherited mental retardation and is characterized by cognitive, neuropsychological, and behavioral
problems. Autistic-like behavior, speech and language delays, anxiety, mental retardation (IQ 35-70), shyness,
attention deficit disorder with or without hyperactivity, and typical facies are observed.
The FBN1 gene codes for the fibrillin-1 protein, and mutation in this gene causes Marfan syndrome. Other
conditions that may arise from the mutation are isolated ectopia lentis, MASS syndrome, etc.Mutation in the KAL
gene on chromosome 22 is the most common cause of Kalmann syndrome. It results in gonadotropin releasing
hormone deficiency along with anosmia or severe hyposomia due to olfactory bulb hypoplasia or agenesis. Findings
include absence of secondary sexual development, anosmia, sensorineural deafness, seizure disorders, decreased
bone density, short metacarpals, and infertility.
Fragile X
The correct answer is fragile X syndrome (FXS).FXS is the most common cause of inherited mental retardation and
is characterized by cognitive, neuropsychological, and behavioral problems. Autistic-like behavior, speech and
language delays, anxiety, mental retardation (IQ 35-70), shyness, attention deficit disorder with or without
hyperactivity, and typical facies are observed. The syndrome results from an expanded CGG repeat (> 200 repeats)
at the 5' untranslated portion of the fragile mental retardation 1 gene (FMR1). This causes a deficiency (or absence)
of the FMR protein. The FMR protein controls the translation of several genes that regulate synaptic plasticity and
development.
Marfan syndrome is a genetic disorder affecting the bodys connective tissue and is caused by mutations in the
FBN1 gene on chromosome 15. Affected individuals are above average height and may have lens subluxation, aortic
dissection, and pneumothorax, apart from other abnormalities.
Williams syndrome is a rare genetic condition with characteristic elflike features, cardiovascular defects,
hypercalcemia, and neurodevelopmental defects. Mental retardation is typically mild to moderate. Language
acquisition is generally delayed; and although quality of language is somewhat normal, language impairment is
present throughout life. Other characteristic features are visuospatial deficits and hypersociability.
Hypothyroidism, growth retardation, and feeding difficulties are additional features. Most cases are sporadic, and
inherited cases are mostly autosomal dominant.
XYY syndrome is a condition in which males have an extra Y chromosome; it is characterized by increased height,
speech delays, and learning difficulties.
Noonan syndrome is a sporadic or autosomal dominant congenital disorder with typical phenotypic features, which
may not be visible to the casual onlooker. The most common facial features include hypertelorism and low-set and
backward rotated ears, along with a thick helix. The philtrum is deeply grooved in more than 90% of cases.
Congenital cardiac defects, bleeding disorders, mental retardation, webbed neck, and short stature are other features.
Autistic Disorder
Describe the 3 categories of symptoms used to diagnose autism.
- Problems w/ social interaction; impairments in communication; repetitive/stereotyped patterns of behavior
or activities
What is the age requirement a/w autism?
- Age 3
(T/F) 70% of individuals w/ autism meet criteria for mental retardation.
- True
Aspergers Disorder
Rett Disorder
What are the characteristics of Rett disorder?

Normal physical and psychomotor development during the first 5 months after birth, followed by a
decreasing rate of head growth and loss of previously learned purposeful hand skills between ages 5 and 30
months
- children then develop stereotyped hand movements (hand wringing/washing), impaired language and
psychomotor retardation, and problems w/ gait or trunk movements
With what gene mutation is Rett disorder associated?
- MECP2 gene mutation on X chromosome
Attention Deficit Hyperactivity Disorder (ADHD)
What are the 3 subcategories of ADHD?
- Predominantly inattentive
- predominantly hyperactive-impulsive
- combined type
Children w/ ADHD have a _____ chance of developing antisocial personality disorder.
- 25%
What are 3 clinical indications for use of atomoxetine in the treatment of ADHD?
- Individuals or families w/ substance abuse problems; individuals w/ tics; patients w/ comorbid anxiety
disorders
Intoxication w/ what heavy metal may lead to hyperactivity which masquerades as ADHD?
- Lead
Name 3 medications used to treat ADHD.
- Dextroamphetamine; methylphenidate; atomoxetine
What is atomoxetine? (Strattera)
- A non-stimulant that has been approved by the FDA for the treatment of ADHD
What drug class is used to treat ADHD should CNS stimulants fail?
- Alpha-2 agonists (clonidine, guanfacine)
Childhood Disintegrative Disorder
What is the age range for development of childhood disintegrative disorder?
- Ages 2-10
Elimination
What are 2 criteria for the diagnosis of encoporesis?
- Greater than 5 years old
- have already been toilet trained
Define enuresis.
- Involuntary voiding of urine (bed-wetting) after age 5 (at least twice a week for at least 3 consecutive
months or w/ marked impairment)
What TCA may be used to treat enuresis?
- Imipramine
Oppositional Defiant Disorder vs. Conduct Disorder
How does oppositional defiant disorder (ODD) differ from conduct disorder?
- ODD does not involve physical aggression or violation of the basic rights of others
Describe the 4 categories of behaviors described by conduct disorder.
- Aggression towards people or animals
- destruction of property
- deceitfulness/theft
- serious violation of rules
What is the diagnostic time period a/w ODD and conduct disorder?
- ODD: 6 months
- conduct disorder: 12 months
Tourettes Disorder
Describe the temporal component of tics required for the diagnosis of Tourette disorder.
- Tics occur may times per day, almost every day for > 1 year (no tic-free period > 3 months)
(T/F) Only motor or vocal tics are required for the diagnosis of Tourette disorder.

- False; both motor and vocal tics must be present to diagnose Tourette disorder
Describe the neurochemical basis of Tourette syndrome.
- Impaired regulation of dopamine in the caudate nucleus
Describe the PANDAS mnemonic and the associated disorder.
- Pediatric Autoimmune Neuropsychiatric Disorders Associated w/ Streptococcal infections
- rapid onset of OCD and/or tic disorders such as Tourette syndrome following group A beta-hemolytic
streptococcal infections
What 2 classes of medications are used to treat Tourette disorder?
- Atypical neuroleptics (e.g. risperidone)
- alpha-2 agonists (e.g. clonidine, guanfacine)
What 2 disorders are often present in patients w/ Tourette syndrome?
- ADHD; OCD
Separation Anxiety Disorder
In what age range is separation anxiety age-appropriate?
- 7 months to 6 years of age
EATING DISORDERS
Anorexia Nervosa
Name the 2 categories of anorexia nervosa.
- Restrictive type
- binge-eating/purging type
Describe the weight requirement(s) listed by DSM-IV for the diagnosis of anorexia nervosa.
- < 85% ideal body weight or BMI < 17.5 kg/m2
What acid-base disturbance is often found in anorexic binge-eating/purging type patients?
- Hypochloremic, hypokalemic alkalosis
What specific characteristic must be present in order to diagnose anorexia nervosa in females?
- Amenorrhea (absence of at least 3 consecutive menstrual cycles)
Bulimia Nervosa
One-fourth of patients w/ bulimia nervosa have comorbid _____.
- Kleptomania
What temporal component is required for diagnosis of bulimia nervosa?
- Binge eating and compensatory behaviors occur at least twice a week for 3 months
According to DSM-IV, what is the most diagnostic feature of bulimia nervosa?
- Recurrent episodes of binge eating
What is the mechanism of sibutramine in the treatment of binge-eating disorder?
- Inhibits reuptake of norepinephrine, serotonin, and dopamine
Describe the required temporal component to bulimia nervosa.
- Binge eating and inappropriate compensatory behaviors both occur, on average, at least twice a week for 3
months
What is the only SSRI approved by the FDA for the treatment of bulimia nervosa?
- Fluoxetine
SOMATOFORM DISORDERS
Name 5 types of somatoform disorders.
- Somatization disorder
- conversion disorder
- pain disorder
- body dysmorphic disorder
- hypochondriasis
Describe the 8 required symptoms for diagnosis of somatoform disorder.
- 4 pain; 2 gastrointestinal; 1 sexual; 1 pseudo-neurological
Somatoform disorder typically presents prior to _____ years of age.

30

Somatization Disorder
What age of onset is required for diagnosis of somatization disorder?
- Onset prior to age 30
What 8 symptoms are required for somatization disorder?
- 4 pain symptoms; 2 gastrointestinal symptoms; 1 sexual/reproductive symptom; 1 pseudoneurological
symptom, not limited to pain
Conversion Disorder
Define globus hystericus.
- The sensation of a lump in the throat experienced by many patients w/ conversion disorder
Pain Disorder
What are 2 subtypes of pain disorder?
- Pain a/w psychological factors
- pain a/w both psychological factors an a general medical condition
Body Dysmorphic Diosrder
Hypochondriasis
One particular form of hypochondriasis, illness phobia, has been reported to respond to _____.
- Imipramine
What is the treatment of choice for body dysmorphic disorder and hypochondriasis?
- SSRIs
(T/F) Hypochondriasis is the only somatoform disorder that does not have a higher frequency in women.
- True
Factitious Disorder vs. Malingering
What distinguishes factitious disorder from malingering?
- Factitious disorder lacks secondary gain
PERSONALITY DISORDERS
Personality disorders cannot be diagnosed before _____ years of age.
- 18.0
Cluster A = Weird; Acusatory, Aloof, Awkward
- Paranoid
What is the treatment of choice for paranoid personality disorder?
- Psychotherapy
- Schizoid
(T/F) There is no increase in incidence of schizoid personality disorder in families w/ history of
schizophrenia.
- True
- Schizotypal
Cluster B = wild; bad to the bone
- Antisocial
What form of psychotherapy has been singularly unhelpful in patients w/ antisocial personality disorder?
- Psychodynamic psychotherapy
This will never be the right answer on the test would fit in dysthymia or personality
disorders, adjustment disorders, GAD. This is the insight oriented therapy; once in a
while MDD.
Development of a _____ disorder becomes more common in patients w/ antisocial personality disorder as
they grow older.
- Somatization

What 2 age-related criteria must be met for the diagnosis of antisocial personality disorder?
- Pattern of disregard for others and violations of the rights of others since age 15; patients must be
at least 18 years old for the diagnosis
What type of therapy is recommended to treat both antisocial and borderline personality disorders?
- Dialectical behavior therapy (DBT)
-

Borderline
What personality disorder is found in up to 70% of patients w/ dissociative identity disorder?
- Borderline personality disorder
What personality disorder presents w/ transient, stress-related psychotic experiences?
- Borderline personality disorder

Histrionic
What defense mechanism is often employed by histrionic patients?
- Regression
Narcissistic

Cluster C = worried; cowardly, compulsive, clingy


- Avoidant
- Dependent
- Obsessive compulsive personality disorder
- Passive Aggressive
COGNITIVE DISORDERS
Name 3 major cognitive disorders.
- Delirium
- Dementia
o Alzheimer Disease
o Vascular Dementia
o AIDS Dementia Complex
- amnestic disorder
o Wernicke-Korsakoff
Delirium
Describe the AEIOU TIPSS mnemonic for causes of delirium.
- Alcohol/drug toxicity
- Electrolyte abnormality
- Iatrogenic
- Oxygen hypoxia
- Uremia/hepatic encephalopathy
- Trauma
- Infection
- Poisons
- Seizures
- Stroke
What condition is described by delirium, tachycardia, tremor, and thyromegaly?
- Thyrotoxicosis
Along w/ delirium, what other 2 symptoms are often present in hypertensive encephalopathy?
- Elevated blood pressure
- papilledema
What class of medications should be avoided in delirious patients?
- Benzodiazepines
What medication can be used to treat anticholingeric delirium caused by TCAs and some neuroleptic drugs?
- Bethanechol

(T/F) As a general rule, olfactory and tactile hallucinations are more common w/ medical conditions rather than
psychosis.
- True
What are the 3 characteristics of Wernicke encephalopathy?
- Delirium; ophthalmoplegia; ataxia
What test is sensitive in determining the etiology of delirium?
- Electroencephalogram (EEG)
What antibiotic is a/w psychosis?
- Isoniazid
Differentiate between hallucinations and illusions.
- Hallucinations: sensory perception that occur in the absence of actual stimulus
- illusions: inaccurate perception of existing sensory stimuli
What 2 drugs should be considered to calm agitated delirious patients?
- Haloperidol IM, PO, IV
- second-generation antipsychotic (e.g. risperidone)
Dementia
What are the "4 A's" of dementia?
- Aphasia
- Amnesia
- Apraxia
- Agnosia
Differentiate between agnosia and apraxia.
- Agnosia: failure to recognize or identify objects despite intact sensory function
- apraxia: impaired ability to carry out motor activities despite intact motor function
Mild memory changes that occur w/ normal aging are called _____.
- Benign senescent forgetfulness
What type of imaging is used to distinguish Alzheimer's from other forms of dementia?
- Fluorodeoxyglucose PET imaging, which shows characteristic temporal and parietal hypometabolism
What is mild cognitive impairment (MCI)?
- Prodromal state in which 12% of patients each year progress to dementia
Name 4 characteristics of Wernicke's encephalopathy.
- Ophthalmoplegia
- ataxic gait
- nystagmus
- mental confusion
What 2 forms of dementia present w/ cogwheel rigidity and resting tremor?
- Lewy body dementia
- Parkinson disease
What endocrinologic disorder can cause reversible dementia?
- Hypothyroidism
Alzheimers Disease
What are 3 genes considered linked to Alzheimer's disease?
- Presenelin I; presenelin II; amyloid precursor protein (APP)
What is the major Alzheimer's susceptibility gene?
- Apolipoprotein e4 (APOe4)
Name 3 cholinesterase inhibitors used to treat Alzheimer's disease.
- Donepezil; rivastigmine; galantamine
_____ is an NMDA antagonist approved for the treatment of moderate-to-severe Alzheimer's disease.
- Memantine
What are 3 anticholinesterase inhibitors used in the treatment of dementia?
- Rivastigmine; galantamine; donepezil
Lewy Body Dementia
What are 4 characteristics of Lewy body dementia?

- Waxing/waning
- Parkinsonism
- visual hallucinations
- sensitivity to neuroleptics
How can Lewy body dementia be differentiated from Parkinson disease dementia?
- Lewy body dementia: onset of dementia within 12 months of parkinsonism symptoms
- Parkinson disease dementia: dementia that begins more than 12 months after the parkinsonism symptoms
What medication is used to treat the REM sleep behavior disorder a/w Lewy body dementia?
- Clonazepam (Klonopin)
Frontotemporal Dementia
Frontotemporal dementia is also known as _____.
- Pick disease
Huntingtons Disease
Describe the pathology of Huntington's disease.
- Trinucleotide repeat (CAG) on short arm of chromosome 4
What MRI finding is characteristic of Huntington's disease?
- Caudate atrophy
(T/F) Dementia due to Parkinson disease is exacerbated by antipsychotic medications.
- True
Describe the pathology a/w Parkinson disease.
- Loss of cells in the substantia nigra of the basal ganglia, causing a decrease in dopamine and loss of the
dopaminergic tracts
Cruetzfeldt Jakob Disease
What symptom is present in over 90% of pts w/ Creutzfeldt-Jakob disease (CJD)?
- Myoclonus (sudden spasms of muscles)
What EEG finding is consistent w/ Creutzfeldt-Jakob disease?
- Triphasic complexes
Normal Pressure Hydrocephalus
Name 3 characteristic symptoms of normal pressure hydrocephalus.
- Ataxia; urinary incontinence; dementia
SUBSTANCE ABUSE AND DEPENDENCE
Substance Abuse/Withdrawal
What 4 criteria define substance abuse?
- Failure to fulfill obligations at work, school, or home
- use in dangerous situations
- recurrent substance-related legal problems
- continued use despite social or interpersonal problems due to the substance use
Opioids
What 2 opioids will not be detected on a standard drug screen?
- Methadone
- oxycodone
What drug combinations are present in Vicodin and Percocet?
- Vicodin: hydrocodone/acetaminophen
- Percocet: oxycodone/acetaminophen
How does meperidine intoxication differ from that of other opioids?
- Meperidine produces mydriasis in contrast to other opioids, which produce miosis
Meperidine and MAOIs taken in combination may cause _____.
- Serotonin syndrome

Sweating, muscle contractions, shivering, CNS symptoms (confusion, disorientation, hypomania,


apprehension, agitation, seizures)
What is the danger of administering naloxone/naltrexone in opioid overdose?
- May cause severe withdrawal in opioid-dependent patients
What drug can be used to treat the autonomic symptoms experienced during opioid withdrawal?
- Clonidine
Alcohol
Describe the effect of alcohol on GABA and glutamate receptors.
- Alcohol activates GABA receptors (inhibitory) and inhibits glutamate receptors (excitatory)
What class of drugs is used to treat acute alcohol withdrawal?
- Benzodiazepines (often chlordiazepoxide)
What electrolyte abnormality may predispose to development of delirium tremens (DTs)?
- Hypomagnesemia
What is the time window for development of DTs?
- 48 - 72 hours
What defines "at-risk" or "heavy" drinking?
- Men: > 4 drinks per day or > 14 drinks per week
- women: > 3 drinks day or > 7 drinks per week
Name 4 medications used to treat alcohol dependence.
- Disulfiram; naltrexone; acamprosate; topiramate
What is the function of topiramate?
- Anticonvulsant that potentiates GABA and inhibits glutamate receptors, reducing the craving for alcohol
What is the major advantage of acamprosate?
- Can be used in patients w/ liver disease
Name 3 characteristics of Wernicke's encephalopathy.
- Broad-based ataxia; confusion; nystagmus
o

PCP
Describe the pharmacologic effect of PCP.
- Antagonizes NMDA glutamate receptors and activates dopaminergic neurons
What symptom is pathognomonic for PCP intoxication?
- Rotary nystagmus
Name 2 serum markers often elevated during PCP use.
- Creatine phosphokinase (CPK)
- aspartate aminotransferase (AST)
What 3 benzodiazepines are indicated in the treatment of PCP intoxication?
- Diazepam; midazolam; lorazepam
Why should typical antipsychotics and benzodiazepines be avoided in PCP intoxication pts?
- Antipsychotics: anticholinergic side effects can worsen side effects
- benzodiazepines: may delay excretion of the drug
BZD/Barbiturates
Differentiate between the effects of benzodiazepines and barbiturates on chloride channel opening.
- Barbiturates: increases the duration of chloride channel opening
- benzodiazepines: increases the frequency of chloride channel opening
What type of drug withdrawal has the highest mortality rate?
- Barbiturate
What short-acting benzodiazepine antagonist can be used to treat benzodiazepine overdose?
- Flumazenil
What is the treatment of choice for opioid overdose?
- Naloxone
Describe the treatment for barbiturate overdose.
- Alkalinize the urine w/ sodium bicarbonate to promote renal excretion
Name 3 benzodiazepines that are not cleared by the liver and may be used in pts w/ chronic alcoholism or liver
disease.

- Lorazepam, oxazepam, temazepam; LOT


Name 2 long-acting (> 20 hrs) benzodiazepines.
- Diazepam; clonazepam
Avoid clonazepam in pts w/ _____.
- Renal dysfunction
What are 4 intermediate-acting (6 - 20 hrs) benzodiazepines?
- Alprazolam; lorazepam; oxazepam; temazepam
Name 2 short-acting (< 6 hrs) benzodiazepines.
Triazolam; midazolam
Marijuana
What is the physiological effect of THC in the brain?
- Activation of cannabinoid receptors, which inhibit adenylate cyclase
Stimulants
What is the pharmacologic effect of cocaine?
- Block dopamine re-uptake from the synaptic cleft, causing a stimulant effect
Describe the pharmacologic effect of amphetamines.
- Block re-uptake and facilitate release of dopamine and norepinephrine from nerve endings, causing a
stimulant effect
Describe the physiologic effect of caffeine.
- Adenosine antagonist, causing an increase in cAMP and a stimulant effect via the dopaminergic system
Name 2 medications used to treat nicotine dependence.
- Varenicline (Chantix)
- buproprion (Zyban)
What is the mechanism of varenicline?
- Alpha-4-beta-2 nicotinic cholinergic receptor (nAChR) partial agonist that mimics the action of nicotine
and prevents withdrawal symptoms
Define formication.
- A hallucinated sensation that insects or snakes are crawling over the skin
- common side effect of extensive use of cocaine and amphetamines
SLEEP
Describe the changes in REM and non-REM sleep seen in the elderly.
- Decreased REM latency and decreased total REM; increased amounts of stage 1/2 sleep and decreased
amounts of stage 3/4 sleep
Describe the half-lives of the non-benzodiazepine hypnotics.
- Zaleplon < zolpidem < eszopiclone
What is the mechanism of the non-benzodiazepine hypnotics?
- Selective receptor binding to benzodiazepine receptor 1
What is ramelteon?
- Selective melatonin MT1/MT2 agonist
Differentiate between dyssomnias and parasomnias.
- Dyssomnias: insufficient, excessive, or altered timing of sleep
- parasomnias: unusual sleep-related behavior
What is the temporal distinction between acute and chronic insomnia?
- Acute: 1 month
What is considered first-line treatment for chronic insomnia?
- CBT
Name the 3 "Z drugs" used to treat primary insomnia.
- Zolpidem (Ambien); eszopiclone (Lunesta); zaleplon (Sonata)
Name 3 antidepressants used to treat primary insomnia.
- Trazodone (most common); amitriptyline; doxepin
What are 3 risk factors for obstructive sleep apnea?
- Obesity
- increased neck circumference

- airway narrowing
Describe the classic tetrad a/w narcolepsy.
- Excessive daytime sleepiness ("sleep attacks")
- REM-related sleep phenomenon including inability to move during the transition from sleep to wakefulness
- hypnagogic/hypnopompic hallucinations
- cataplexy
What is cataplexy?
- Sudden loss of muscle tone evoked by strong emotion w/o LOC
What is the drug of choice to treat cataplexy in narcoleptic pts?
- Sodium oxybate
Define Kleine-Levin syndrome.
- Rare disorder characterized by recurrent hypersomnia w/ episodes of daytime sleepiness w/ hyperphagia,
hypersexuality, and aggression
Name 4 circadian rhythm sleep disorders.
- Delayed sleep phase disorder
- advanced sleep phase disorder
- shift-work disorder
- jet lag disorder
Describe image rehearsal training (IRT).
- Involves the use of mental imagery to modify the outcome of a recurrent nightmare, writing down the
improved outcome, and then mentally rehearsing it in a relaxed state
REM Sleep Disorder
Characterize REM sleep behavior disorder.
- Characterized by muscle atonia during REM sleep and complex motor activity a/w dream mentation
(dream enactment)
What 2 types of dementia are a/w REM sleep behavior disorder?
- Olivopontocerebellar atrophy
- Lewy body dementia
What benzodiazepine is efficacious in 90% of pts w/ REM sleep behavior disorder?
- Clonazapam
RANDOMNESS
What 4 dissociative disorders are described in DSM-IV?
- Dissociative amnesia
- dissociative fugue
- dissociative identity disorder
- depersonalization disorder
What condition has the worst prognosis of all the dissociative disorders?
- Dissociative identity disorder
What are the 3 criteria for delusional disorder?
- Nonbizarre, fixed delusions for at least 1 month
- does not meet criteria for schizophrenia
- functioning in life not severely impaired
What is selective mutism?
- Rare condition characterized by refusal to speak in certain situations (e.g. school) for at least 1 month
despite the ability to comprehend and use language
What is Ganser syndrome?
- The giving of approximate answers to simple questions
What is ataque de nervios?

Culturally-bound trance disorder common in Puerto Rico that consists of convulsive movements, fainting,
crying, and visual problems

What are the 5 stages of normal sexual response?


- Desire; excitement; plateau; orgasm; resolution
What are the 3 most common paraphilias?
- Pedophilia; voyeurism; exhibitionism
The most important factor in assessing a patient's risk of violence is _____.
- Individual's history of violence
Describe reaction formation.
- A defense mechanism by which an individual deals w/ emotional conflict or stressors by substituting
behavior, thoughts, or feelings that are diametrically opposed to his or her own unacceptable thoughts or
feelings
Medication Adverse Reactions
What are HAM side effects?
- antiHistamine (sedation, weight gain)
- antiAdrenergic (hypotension)
- antiMuscarinic (dry mouth, blurred vision, urinary retention)
HAM side effects are found w/ what 2 classes of psychiatric medications?
- TCAs; low-potency antipsychotics
Extra-Pyramidal Side Effects
What is the drug of choice to treat EPS produced by neuroleptics?
- Benztropine
Describe 3 forms of EPS.
- Parkinsonism (masklike face, cogwheel rigidity, pill-rolling tremor)
- akathisia (restlessness, agitation)
- dystonia (sustained contraction of muscles of neck, tongue, eyes, diaphragm)
Name 3 medications that may be used to treat EPS.
- Benztropine
- Diphenhydramine
- amantadine
What scale is used to quantify and monitor for tardive dyskinesia?
- Abnormal Involuntary Movement Scale (AIMS)
What medication may be used to treat akathisia?
- Propranolol
Neuroleptic Malignant Syndrome
What are 2 characteristic features of neuroleptic malignant syndrome?
- Elevated creatine phosphokinase (CPK)
- "lead pipe" rigidity
What 2 medications are commonly used to treat NMS?
- Dantrolene
- bromocriptine
Serotonin Syndrome
Describe the neuromuscular excitability a/w serotonin syndrome.
- Hyperreflexia; "electric jolt" limb movements
What class of medications may be useful in treating serotonin syndrome?
- Calcium channel blockers (e.g. nifedipine)
Wait at least _____ before switching from SSRI to MAOI, and at least _____ w/ fluoxetine.

2 weeks; 5 - 6 weeks

Drug Interactions
Name 4 important inducers of the CYP450 enzyme.
- Smoking (1A2)
- carbamazepine (1A2, 2C9, 3A4)
- barbituates (2C9)
- St. John's wort (2C19, 3A4)
What are 5 important inhibitors of the CYP450 enzyme?
- Fluvoxamine (1A2, 2D6, 3A4)
- fluoxetine (2C19, 2C9, 2D6)
- paroxetine (2D6)
- duloxetine (2D6)
- sertraline (2C19)
SSRIs
What SSRI has a weekly dosing form available?
- Fluoxetine
What SSRI has the highest risk for GI disturbances?
- Sertraline
Name 7 SSRIs.
- Fluoxetin; sertraline; paroxetine; fluvoxamine; citalopram; escitalopram
What SSRI is currently only approved for the treatment of OCD?
- Fluvoxamine
What SSRI is preferred in the treatment of intermittent explosive disorder?
- Fluoxetine
Low levels of what neurotransmitter is a/w impulsiveness and aggression?
- Serotonin
What class of SSRI-related side effects may occur later in the treatment course?
- Sexual dysfunction poor libido, anorgasmia
SNRIs
Name 2 SNRIs.
- Venlafaxine; duloxetine
What are 3 clinical indications for venlafaxine?
- Depression
- anxiety disorders (e.g. GAD)
- ADHD
Duloxetine is often used for patients w/ depression and _____.
- Neuropathic pain
What contraindication is a/w venlafaxine?
- May increase BP; do not use in pts w/ untreated for labile HTN
NDRI
Describe the mechanism of bupropion.
- Norepinephrine-dopamine reuptake inhibitor
it is so stimulating, Ive had patients have spontaneous orgasms on it.
Stimulating, seizure threshold.
Other Antidepressants
What are 3 clinical indications for trazodone and nefazodone?
- Refractory major depression; major depression w/ anxiety; insomnia
What major adverse effect is a/w trazodone? Nefazodone?
- Trazodone: priaprism
- nefazodone: liver failure
Describe the mechanism of mirtazapine.

2-adrenergic receptor antagonist


o first SNRI to enter the market as opposed to Prozac minor weight gain, sedation
o all of the SSRIs and all of the SNRIs amphetamines, topiramate, and bupropion dont cause
weight gain.

Tricyclic Antidepressants
Describe the mechanism of tricyclic antidepressants.
- Inhibit reuptake of norepinephrine and serotonin, increasing availability of monoamines in the synapse
Name 7 tricyclic antidepressants and their subcategories.
- Tertiary amines: amitriptyline, imipramine, clomipramine, doxepin; secondary amines: nortriptyline,
desipramine
What TCA may be used to treat insomnia?
- Doxepin
What are 2 clinical indications for use of imipramine?
- Enuresis; panic disorder
What TCA has the least anticholinergic side effects?
- Desipramine
What TCA is least likely to cause orthostatic hypotension?
- Nortriptyline
The mainstay of treatment for TCA overdose is _____.
- IV sodium bicarbonate
Name 2 tetracyclic antidepressants.
- Amoxapine; maprotiline
What are the "3 C's" of TCA side effects?
- Cardiotoxicity
- Convulsions
- Coma
What tricyclic antidepressant is preferred for use in the elderly population?
- Nortriptyline, as it has the fewest side effects
TCA absolute contraindication to TCA is glaucoma can exacerbate closed angle decrease reabsorption; 1015% of glaucoma is narrow angle.
Mono-Amine Oxidase Inhibitors
Describe the mechanism of MAOIs.
- Prevent the inactivation of biogenic amines such as norepinephrine, serotonin, dopamine, and tyramine by
irreversibly inhibiting the MAO-A/B enzymes
Describe the selectivity of MAO-A and MAO-B enzymes.
- MAO-A preferentially deactivates serotonin and MAO-B preferentially deactivates norepinephrine; both
types also act on dopamine and tyramine
MAOIs are considered more effective in treating _____ depression, characterized by hypersomnia, increased
appetite, and increased sensitivity to interpersonal rejection.
- Atypical
Name 3 MAOIs.
- Phenelzine; tranylcypromine; isocarboxazid
What is the most common side effect of MAOIs?
- Orthostatic hypotension
Antipsychotics
Quetiapine sedating
Name 6 atypical antipsychotic medications.
- Clozapine; risperidone; quetiapine; olanzapine; ziprasidone; aripiprazole
_____ is the only antipsychotic that has been proven to decrease the risk of suicide.
- Clozapine
(T/F) 30% of treatment-resistant psychosis will respond to clozapine.
- true
Differentiate between the mechanism of typical and atypical antipsychotics.

- Typical: block dopamine (D2) receptors; atypical: block both dopamine (D2) and serotonin (2A) receptors
(T/F) Atypical antipsychotics may be more effective in treating negative psychotic sx, such as flattened affect and
social withdrawal.
- true
Name 2 low potency typical antipsychotic medications and the pigment-related side affects a/w each.
- Chlorpromazine (bluish skin discoloration)
thioridazine (retinitis pigmentosa)
_____ may be used to treat intractable hiccups.
- Chlorpromazine
Why are low potency typical antipsychotics more lethal in overdose than mid- and high-potency typical
antipsychotics?
- QTc prolongation and potential for heart block and ventricular tachycardia
Name 4 mid-potency typical antipsychotics.
- Loxapine; thiothixene; trifluoperazine; perphenazine
Name 3 high-potency typical antipsychotics. developed for the treatment of psychosis
- Haloperidol; fluphenazine; pimozide
What 2 first-generation antipsychotic medications are a/w ocular abnormalities?
- Thioridazine: irreversible retinal pigmentation
- chlorpromazine: deposits in lens and cornea
A risk of agranulocytosis, requiring weekly prophylactic blood draws, is a/w _____.
- Clozapine
What 2 atypical antipsychotic medications are most a/w metabolic syndrome?
- Clozapine; olanzapine
o Olanzapine = causes WEIGHT gain average of 25 lbs weight gain.
What atypical antipsychotic is considered "weight neutral"?
- Ziprasidone
What atypical antipsychotic is a/w elevated prolactin levels?
Risperidone
Differentiate between the mechanism of typical and atypical antipsychotics.
- Typical: block dopamine (D2) receptors
- atypical: block both dopamine (D2) and serotonin (2A) receptors
(T/F) Atypical antipsychotics may be more effective in treating negative psychotic sx, such as flattened affect and
social withdrawal.
- true
Mood Stabilization
Name 4 mood stabilizers.
- Lithium; valproic acid; lamotrigine; carbamazepine
Describe the therapeutic, toxic, and lethal ranges of lithium.
- Therapeutic: 0.6 - 1.2; toxic: > 1.5; lethal: > 2.0
Lithium thyroid function tests to get baseline, EKG can exacerbate existing arrhythmias, kidney function
KNOW lithium b/c it is the gold standard
Most of the time you are starting lithium on manic patient.
What is the effect of concurrent NSAID use w/ lithium?
- increased lithium levels
What is the only mood stabilizer w/ FDA approval for treatment of bipolar disorder in children older than 12?
- Lithium
Lithium Side Effects:
- weight gain, tremor, GI disturbances (anal ulcers, lip lesions, N/V, anything in between), fatigue,
arrhtyhmias, seizures, goiter/hypothyroidism, leukocytosis (benign) may be used to increase WBC in
clozapine users, coma, polyuria, polydipsia, alopecia, metallic taste
- Acute: nausea with or without vomiting, fine tremor, polyuria, mild confusion, t-wave EKG changes,
muscle weakness
o Then: coarse tremor, ataxia, confusion, slurred speech, persistent nausea and diarrhea
o Then: convulsions, hypotension, irregular heart beat, coma

Chronic: nephrogenic DI, hypothyroidism/goiter, weight gain, edema, rashes and aggravation of skin
conditions (acne, psoriasis), benign leukocytosis, teratogen (ebsteins anomaly)

In cases of bipolar disorder where the use of lithium is contraindicated, what is the drug of choice?
- Valproic acid
Describe the clinical indication for use of carbamazepine.
- Especially useful in treating mixed episodes and rapid-cycling bipolar disorder, and less effective for the
depressed phase
Describe the mechanism of carbamazepine.
- Acts by blocking sodium channels and inhibiting action potentials
What are 2 catastrophic side effects of carbamazepine?
- Stevens-Johnson syndrome
- teratogenic effects when used during pregnancy
Describe the mechanism of lamotrigine.
- Effective on sodium channels that modulate glutamate and aspartate
Lamotrigine is most efficacious for treating _____.
- Bipolar depression (though little efficacy for acute mania or prevention of mania)
Name 3 side effects of topiramate.
- Weight loss; hypochloremic non-anion gap metabolic acidosis; kidney stones
Legal
What are the "4 D's" of malpractice?
- Deviation (neglect) from Duty that was the Direct cause of Damage
In what 3 situations is informed consent not required to treat unemancipated minors?
- Obstetric care
- STD treatment
- substance abuse treatment
What are the "4 R's" of informed consent?
- Reason for treatment
- Risks and benefits
- Reasonable alternatives
- Refused treatment consequences
Under what 4 conditions are minors considered emancipated?
- Self-supporting
- in the military
- married
- have children
Differentiate between capacity and competence.
- Capacity: clinical term and may be assessed by physicians
- competence: legal term and can be decided only by a judge
(T/F) Decisional capacity is task specific and can fluctuate over time.
- true
What is Parens Patriae?
- Legal principle that supports involuntary commitment of citizens who cannot care for themselves
What is the M'Naghten test?
- Most stringent test used as a standard of the insanity defense; indicates person does not understand what
he/she was doing or its wrongfulness

DSM-IV diagnostic Criteria


Psychotic Disorders
- Schizophrenia 2 or more for at least 1 month with total disease > 6
months causing significant social/occupational functional deterioration
o delusions, hallucinations, disorganized speech, grossly disorganized or
catatonic behavior, negative symptoms (anhedonia, flattened or
inappropriate affect, alogia (poverty of speech), avolition
(apathy), attention poor)
when she writes on schizophrenia, she writes on 4As.
Ambivalence
Allusions (delusions/hallucinations)
Affect flattened or inappropriate
Association (tangentiality)
o tx 4 weeks before efficacy determined
- Brief psychotic disorder - < 1 month
- Schizophreniform 1-6 months
- Schizoaffective disorder major depressive, manic, or mixed episode +
delusions or hallucinations for 2 weeks in the absence of mood d/o sx.
o Tx: antipsychotics, mood stabilizers, antidepressants, ECT for
depression/mania
- Delusional Disorder non-bizarre, fixed delusions for > 1 month, but does
not meet criteria for schizophrenia and functioning in life is not significantly
impaired.
- Shared Psychotic Disorder tx: separate; psychotherapy + antipsychotics if
sx do not improve 1-2 weeks after separation
- Koro asia penis shrinking and will disappear = death
- Amok Malaysia, SE Asia sudden unprovoked outbursts of violence w/ no
recollection often commits suicide afterwards
- Brain Fag Africa HA, fatigue, visual disturbances in male students
Mood Disorders (symptoms with depression as a symptom)
- Major Depressive Disorder:
o At least one episode of Major Depression: 5/9 sx w/ one being
depression or anhedonia for at least 2 weeks. Sleep, psychomotor,
appetite, concentration, energy, depression, interest, guilt, suicidal
ideations
o Without mania/hypomania
o With psychotic features ECT alone (better when rapid response is
needed) or antidepressant + antipsychotic
- Bipolar I
o One manic or mixed episode (depression NOT required for dx)
Mania = 3 of the following for at least 1 week: distractability,
insomnia, grandiosity, flight of ideas, activity/agitation, speech
pressured, thoughtlessness (or 4, with irritable mood)
Mixed = mania/depression sx present for at least 1 week
- Bipolar II
o Recurrent major depressive episodes with HYPOmania

Hypomania at least 4 days; no marked impairment in social or


occupational functioning, no psychotic features; does not require
hospitalization
Dysthymic Disorder 2 or more of the following symptoms for at least 2
years (or 1 year in children), not without symptoms for > 2 months at a
time, with no major depressive episode
o CHASES concentration, hopelessness, appetite (+/-), sleep (+/-),
Energy low, self-esteem low
o NEVER have psychotic features
Double Depression MDD with dysthymic d/o during residual periods. Tx:
cognitive therapy and insight oriented psychotherapy; antidepressants
concurrently
Cyclothymic Disorder alternating periods of hypomania/mild-moderate
depressive sx.
o Numerous periods of hypomania/depressive sx for > 2 years with no
asx periods > 2 months and no hx of major depressive or manic
episodes.
minor depressive disorder depressive sx that do not meet criteria for MDD,
with euthymic periods (unlike dysthymic disorder)
Adjustment disorder with depressed mood
Secondary to a medical disorder
o MSE: affect labile (sudden rapid shifts congruent with mood), TP
irrelevant, TC frank visual hallucinations (not
hypnogognic/hypnopompnic) have a metabolic, toxic or structural
property unless proven otherwise. The one exception visual
hallucinations of knee-high men (I see little men, sometimes
green) = malingering, Intellectual function concentration is often
the first thing to go and the rest of the exam is questionable
Secondary to substance disorder

MSE: TP irrelevant = depression secondary to medical disorder, loosening and


tangentiality and concretism
ANXIETY AND ADJUSTMENT DISORDERS
- Panic Disorder at least 1 Spontaneous, recurrent panic attack followed by at
least 1 month of the following: worrying about recurrence, worry about
implications of the attack, significant change in behavior related to attack
o Panic attack = intense fear and discomfort plus 4 of the following
symptoms that peaks in several minutes; subside w/i 25 min. (1020 minutes maximum for chest pain/pulmonary) rarely > 1 hr
Palpitations
Abdominal distress
Numbness, Nausea
Intense fear of death; sense of impending doom
Choking, Chills, Chest pain
Sweating, Shaking, SOB
o Rule Out:
Asthma

Breathing - COPD
Cardiac (MVP, arrhythmia, CHF)
Drugs (cocaine, amphetamines, caffeine, nicotine, hallucinogen
intoxication; alcohol or opiate withdrawal)
Endocrine (thyroid, pheochromocytoma)
Agoraphobia anxiety about being in places/situations where escape may be
difficult; situations are either avoided, endured with distress, or faced only
with companion; sx not otherwise explainable. Tx: SSRI
Specific Phobias
o Persistent excessive fear brought on by specific situation/object where
exposure brings immediate anxiety, pt recognizes fear is excessive,
situation is avoided when possible, and if person < 18 yo, sx for at
least 6 months
Tx: systematic desensitization and supportive psychotherapy
Straight behavioral therapy = flooding or desensitization
Cognitive behavioral therapy = not either of these. USE
FOR DEPRESSION
BZD or beta blockers (short course)
Social Phobias same as specific w/ fear related to social setting where
embarrassment or humiliation may occur.
o Paroxetine = social anxiety disorder
Obsessive Compulsive Disorder obsessions and compulsions that the person
is aware are unreasonable and excessive; cause marked distress, are time
consuming, or significantly interfere with daily functioning. Tx: SSRI,
clomipramine, behavioral therapy (exposure and response prevention)
PTSD response to life-threatening traumatic experience where patient reexperiences the trauma, avoids reminders of the event, and experiences
emotional numbing or hyperarousal (difficulty sleeping, outbursts of anger,
exaggeurated startle response, difficulty concentrating) with symptoms
present for at least 1 month and may occur any time after experience
Acute Stress Disorder major life-threatening traumatic event but anxiety
symptoms for only a short duration: event occurred < 1 month ago and
symptoms last < 1 month
Generalized Anxiety Disorder excessive persistent anxiety about daily
events and activities with hyperarousal for at least 6 months
o Must be associated with 3 or more of the following: restlessness,
fatigue, difficulty concentrating, irritability, sleep disturbance, muscle
tension
o Tx: buspar, BZD (should be tapered off ASAP), SSRI +/- CBT,
venlafaxine
o Free floating anxiety does not involve any specific person, event, or
activity
Adjustment Disorders development of emotional or behavioral symptoms
within 2 months after a stressful but non-life threatening event (severe
distress in excess of what would be expected after such an event with
significant impact in daily functioning and sx are NOT those of bereavement.
o Sx resolve within 6 months of termination of stressor
o Tx: supportive psychotherapy most helpful; group therapy;
pharmacotherapy for associated symptoms

PERSONALITY DISORDERS
Cluster A: Accusatory, aloof, awkward
- Paranoid 4 or more of the following:
o Suspicion w/o evidence of exploitation/deception
o preoccupation with doubts of loyalty or trustworthiness
o Reluctance to confide in others
o Interpretation of benign remarks as threatening or demeaning
o Persistence of grudges
o Perception of attacks on his/her character that are not apparent to
others/quick to counterattack
o Recurrence of suspicions regarding fidelity of spouse/lover
- Schizoid no desire for close relationships - 4 or more of the following
o Neither enjoying nor desiring close relationships
o Generally chosing solitary activities
o Little interest in sexual activity
o Taking pleasure in few activities
o Few close friends or confidants
o Indifference to praise or criticism
o Emotional coldness, detachment, or flattened affect
- Schizotypal magical thinking, eccentric behavior, peculiar thought
processes. 5 or more of the following:
o Ideas of reference
o Odd beliefs or magical thinking, inconsistent with cultural norms
o Suspiciousness
o Inappropriate or restricted affect
o Odd or eccentric appearance/behavior
o Few close friends or confidants
o Odd thinking or speech (vague, stereotyped)
o Excessive social anxiety
Cluster B: Bad
- Antisocial pattern of disregard and violation of rights of others since age
15, and patient is at least 18 years old. (R/O drug abuse difficult to
determine which came first); Begins in childhood as conduct disorder. 3 or
more of the following should be present:
o Failure to conform to social norms by committing unlawful acts
o Deceitfulness/repeated lying/manipulating others for personal gain
o Impulsivity/failure to plan ahead
o Irritability and aggressiveness/repeated fights or assaults
o Recklessness and disregard for safety of others and self
o Irresponsibility/failure to sustain work or honor financial obligations
o Lack of remorse for actions
- Narcissistic superiority, need for admiration; consider themselves special
and will exploit others for personal gain, but fragile self esteems;
antidepressants or lithium may be used as needed if co-morbid mood d/o
exists; and at least 5 of the following:
o Exaggerated sense of self-importance

Preoccupation with fantasies of unlimited money, success, brilliance


Belief that he or she is special or unique and can associate only with
other high-status individuals
o Needs excessive admiration
o Sense of entitlement
o Takes advantage of others for self-gain
o Lacks empathy
o Envois or others or believes others are envious of him/her
o Arrogant or haughty
Histrionic Tx: psychotherapy, pharmacotherapy for depressive/anxiety sx as
necessary attention-seeking, flamboyant behavior unable to form longlasting, meaningful relationships with at least 5 of the following:
o Uncomfortable when not the center of attention
o Inappropriately seductive or provocative
o Uses physical appearance to draw attention to self
o Has speech that is impressionistic and lacking in detail
o Theatrical and exaggerated expression of emotion
o Easily influence by others or situation
o Perceives relationships as more intimate than they actually are
Borderline on the borderline of neurosis and psychosis Tx: DBT, with
pharmacotherapy to treat psychotic and depressive symptoms as necessary
(low dose antipsychotic or SSRI pharmacotherapy more effective in BPD
than any other PD) - at least 5 of the following must be present:
o Impulsive in at least 2 harmful ways (spending, sexual activity,
substance abuse)
o Moody/unstable affect
o Paranoid under stress transient stress-related paranoid ideation or
dissociative symptoms
o Unstable body image
o Labile, intense relationships
o Suicidal recurrent threats or self-mutilation
o Inappropriate anger
o Vulnerable to abandonment
o Emptiness
o
o

Cluster C: cowardly, compulsive, clingy


- Avoidant desire companionship but are extremely shy and easily injured;
cling to relationships but SLOW to get involved; 4 or more of the following:
o Avoids occupation that involves interpersonal contact due to fear of
criticism or rejection
o Unwilling to interact unless certain of being liked
o Cautious of interpersonal relationships
o Preoccupied with being criticized or rejected in social situations
o Inhibited in new social situations because he or she feels inadequate
o Believes he or she is socially inept or inferior
o Reluctant to engage in new activities for fear of embarrassment.
- OCPD egosyntonic; motivated by work itself, vs. narcissistic who are
motivated by status. preoccupation with orderliness, control, and

perfectionism at the expense of efficiency, present in early adulthood in a


variety of contexts. at least 4 of the following:
o Preoccupation with details, rules, lists, and organization such that the
major point of the activity is lost
o Perfectionism that is detrimental to the completion of task
o Excessive devotion to work
o Excessive conscientiousness and scrupulousness about morals and
ethics
o Will not delegate tasks
o Unable to discard worthless objects.
o Miserly
o Rigid and stubborn.
Dependent must manifest by early adulthood. cling to relationships, but
actively and aggressively seek relationships; Submissive and clinging
behavior with excessive need to be cared for with at least 5 of the following:
o Difficulty making everyday decisions without reassurance from others
o Needs others to assume responsibilities for most areas of life
o Cannot express disagreement due to fear of loss of approval
o Difficulty initiating projects due to lack of self-confidence
o Goes to excessive lengths to obtain support from others
o Feels helpless when alone
o Urgently seeks another relationship when one ends
o Preoccupation with fears of being left to take care of self.

COGNITIVE DISORDERS
- Cortical Dementias = decline in intellectual functioning
o Alzheimers memory impairment plus at least 1: apraxia, aphasia,
agnosia, diminished executive functioning
Apraxia = cant do Practiced movements even though motor
system is intact
Aphasia cant speek/understand phrases
Agnosia cannot identify even though sensory system is intact
o Multiinfarct = stepwise increase in severity, focal neuro signs
Memory impairment plus at least 1: apraxia, aphasia, agnosia,
diminished executive functioning
Personality changes (depression, anger, suspiciousness
common), but greater overall preservation of personality
Paranoia also common
o Picks/Frontotemporal Dementia slowly progressive
Aphasia, apraxia, agnosia
Personality and behavior changes are more prominent early in
disease vs. alzheimers. Disinhibited or more apathetic.
Atrophy of frontotemporal lobes, pick bodies (intraneuronal
inclusion bodies)
o CJD
Rapidly progressive dementia (6-12 months) after onset of
symptoms with myoclonus. May also have EPS, ataxia, cortical
blindness, muscle atrophy, and mutism

Subcortical dementias prominent affective and movement symptoms.


o Huntington Disease = choreiform movement, 35-50 yo, AD,
hypertonicity, depression, psychosis, CAG repeat on Ch4, MRI with
caudate atrophy
o LBD/PD = cogwheel rigidity, resting tremor
PD: bradykinesia, resting (pill rolling) tremor, cogwheel rigidity,
masklike facial expression, shuffling gait, dysarthria (abnormal
speech)
LBD: visual hallucinations, REM sleep disturbances, PD
symptoms,
o NPH = ataxia, urinary incontinence, dilated ventricles
Due to trauma, infection, hemorrhage. Tx with shunt
o Hypothyroidism = obesity, coarse hair, constipation, cold intolerance
o Vitamin B12 deficiency = diminished position/vibratory sense,
megaloblasts
o Wilsons Disease = tremor, abnormal LFT, kayser-fleischer rings
o Neurosyphilis = diminished position/vibratory sense, Argyll-robertson
pupils
Delirium detect with EEG (very sensitive to slowing of the waves)
o Causes of Delirium = IM DELIRIOUS
Impaired delivery of brain substrates (vascular insufficiency
stroke)
Metabolic
Drugs
Endocrinopathy
Liver disease
Infrastructure (structural disease of cortical neurons)
Renal failure
Infection
Oxygen
UTI
Sensory deprivation
o After cardiac surgery age, alcohol, brain damage, diabetes
o CVA or mass lesion delirium + hemiparesis or other focal neuro ssx
o HTN encephalopathy elevated BP, papilledema, delirium
o Drug Intoxication dilated pupils, tachycardia, delirium
o Meningitis nuchal rigidity, photophobia, fever, delirium
o Thyrotoxicosis tachycardia, tremor, thyromegaly delirium

GERIATRIC
- Pseudodementia
o Apparent cognitive deficits in pts with major depression.
o Onset is acute, patient emphasizes failures (vs. dementia where
delights in accomplishments), sundowning uncommon (vs. dementia
where common), often answers Dont know (vs. confabulation in
dementia), aware of problems (vs. unaware in dementia)
- Bereavement

o
o

Normal grief guilt and sadness, mild sleep disturbances and weight
loss, illusions, attempts to resume daily activities/work, symptoms that
resolve within 1 year, with the worse symptoms within 2 months
Abnormal grief (major depression) feelings of severe guilt or
worthlessness, significant sleep disturbances or weight lsos,
hallucinations or delusions, no attempt to resume activities, suicide
ideation, persisting > 1 year with worse symptoms > 2 months

CHILDREN have a lot of anxiety d/o, not as many mood d/o.


- Mental Retardation Causes:
o Down Syndrome
o Anoxia (perinatal)
o Prematurity
o Birth trauma
o Hypothyroidism
o Malnutrition
o Toxin exposure
o trauma
o Fragile X
o TORCH infections and toxins
- Learning disorders
o Achievement in reading (boys), math (girls), or written expression
lower than expected for age. Tx: remedial education
- Pervasive developmental disorders
o Autism kids appear to be developing normal until after 2
At least 6 symptoms must be present:
at least 2 problems with social interaction
at least 1 impairment in communication
at least 1 repetitive and stereotyped pattern of behavior
and activities
o Asperger
Impaired social interaction (at least 2) and restricted or
steortyped behaviors, interests, or activites BUT NORMAL
COMMUNICATION and cognitive development
o Rett
Normal prenatal and perinatal development, psychomotor
development during first 5 months of life, normal initial head
circumference with decreasing rate of head growth between 548 months loss of purposeful hand skills between 5-30
months with handwringing, handwashing, loss of social
interaction, probelmsw ith gait or trunk mvmts, seizures,
cyanotic spells, and severely impaired language and
psychomotor development
o Childhood disintegratice disorder
Normal development in first 2 years of life with loss of
previously acquired skills in at least two of the following areas:
language, social skills, bowel or bladder control, play or motor
skills. Also, at least two of the following: impaired social

interaction, impaired use of language, or restricted, repetitive,


and stereotyped behaviors and interests.
MORE COMMON IN BOYS! (4-8xs)
Disruptive behavioral disorders
o Conduct disorders by self
Major depression from conduct d/o in kids
Failing in schools, stealing cars conduct disorder acting
out in 2 places = conduct disorders
Doing find in school, very obedient, stealing cars
depression (acting out in one place = depression)
Violation of basic rights of others or social norms and rules with
at least 3 acts of:
Aggression towards people or animals
Destruction of property
Deceitfulness/theft
Serious violations of rules
Tx: structure environment; pharmacotherapy (SSRI, lithium),
behavior modification
o Oppositional Defiant disorder after being told not to do something;
something to which opposition is being demonstrated
at least 6 months of negativistic, hostile, or defiant
behavior during which at least 4 of the following are present:
frequent loss of temper
arguments with adults
defying adults rules
deliberately annoying people
easily annoyed
anger and resentment
spiteful
blaming others for mistakes or misbehaviors
Tx: behavior modification and problem solving skills
ADHD with and without hyperactivity know classes
o Inattnetive, hyperactive-impulsive, OR combined type.
o At least 6 symptoms persisting for at least 6 months presenting
before age 7
Inattention:
Problems listening, concentrating, paying attention to
details, organizing tasks
Easily distracted, often forgetful.
Hyperactivity/impulsivity:
Blurting out, interrupting, fidgeting, leaving seat, talking
excessively
Tourette/Tic
o Involuntary mvmt or vocalizations with tics occurring multiple times
a day almost everyday for > 1 year with no tic free period > 3
months, with onset prior to age 18 and distress and impairment in
social/occupational functioning

Elimination
o Enuresis involuntary voiding after age 5, which occurs at least twice
a week for 3 months or with marked impairment.
o Encopresis involuntary or intentional passage of feces in
inappropriate places; must be at least 4 years old with at least once
a month episodes for 3 months
Selective Mutism
o Not speaking in certain situations (school), with onset usually 5-6
years old and more common in girls. Present for at least 2 months
KNOW normal child cling to child? Normally messy?

DISSOCIATIVE
- Dissociative Amnesia at least one episode of being unable to recall
important personal information, usually involving traumatic or stressful event,
which cannot be explained by ordinary forgetfulness, and symptoms cause
distress or impairment in daily functioning (cannot be explained by another
disorder, medical condition, or substance use). (often unable to recall name
but will remember obscure details, vs. dementia pt who recalls name but not
details)
- Dissociative Fugue sudden, unexpected travel away from home or work plus
the inability to recall ones past, with confusion about personal identity or
assumption of new identify, NOT due to dissociative identity disorder or
physiologic effects of a substance/medical disorder. Impairment in social or
occupational fxn.
- Dissociative Identity Disorder at least two or more distinct identities that
recurrently take control of a persons behavior with the inability to recall
personal information of one personality when the other is dominant; not due
to substance use/medical condition
o WORST prognosis of all dissociative disorders
- Depersonalization disorder
o Persistent or recurrent experiences of being detached from ones body
or mental processes, but reality testing remains intact during episode;
social or occupation impairment hat cannot be accounted for by
amouther mental or physical disorder
Tx: retrieve lost memories using hypnosis, sodium amobarbital or lorazepam during
the interview.

PRACTICE QUESTIONS
Q1) A 4-year-old boy has just been reunited with his biological mother after spending 6 months in foster care. He
was initially removed from his home for severe neglect secondary to substance abuse. The boy is noted to play in a
destructive and disorganized manner, throwing and breaking his toys. When the mother tries to stop him from
throwing blocks at her, he starts kicking and biting. Which of the following is the most likely diagnosis?
a) Oppositional defiant disorder (ODD)
b) ADHD
c) Reactive attachment disorder
d) PTSD
e) Major depression
Q2) Secondary prevention is best exemplified by which of the following?
a) Enrollment of a 2 year-old in a play group in the hopes of preventing antisocial behaviors.
b) Restraing a violent teen after a suicide attempt.
c) Visiting the home of a child after a neighbor reports that the child is being neglected.
d) Treating new onset feelings of depression and anhidonia with an SSRI.
e) Enrolling siblings of bipolar patients in group therapy.
Q3) A 56-year-old man is brought to your office by his wife because she has noted a personality change in the past 3
months. While the patient is being interviewed, he continually discusses event on the news, what he ate for lunch
and other topics unrelated to the questioning prior to providing appropriate answers to your questions. Which of the
following symptoms best fits this patient's behavior?
a) Negative symptoms
b) Disorientation
c) Concrete thinking
d) Perseveration
e) Circumstantiality
1. A 3-year-old girl is brought to the physician by her parents
because they are concerned about her behavior. They describe
their daughter as stubborn and always on the go. She can
rarely sit still for more than 10 minutes. She often refuses to
comply with their requests and sometimes throws 3- to 5minute temper tantrums. They report that she dawdles at
bedtime and requires frequent direction and assistance in
preparing for bed. Her preschool teacher notes that she is
active and talkative without being disruptive and is beginning
to demonstrate more interactive play with her peers. She
generally sleeps through the night and occasionally wets the
bed. Her appetite is good. Her first word was at the age of 11
months, and she began walking without assistance at the age
of 14 months. Physical examination shows no abnormalities.
On mental status examination, she initially hides behind her
mother but warms to the interviewer after a few minutes and
begins playing with toys in the office. Her speech is 90%
intelligible, and her vocabulary is large for her age. Which of
the following is the most appropriate next step in
management?
(A) Reassurance
(B) Play therapy
(C) Speech therapy
(D) Enuresis alarm
(E) Trial of fluoxetine
(F) Trial of methylphenidate

2. A 45-year-old man is brought to the physician by his spouse.


He has been drinking heavily since he was passed over for a
job promotion 3 days ago. He stayed in bed over the
weekend. He has no personal history of psychiatric disorders
and no personal or family history of alcohol abuse. He is
crying and states, AI can't believe it,@ when addressed. When
asked what he will do, he states, AI don't know, but if I don't
go back to work tomorrow, I'll lose my job.@ Which of the
following is the most likely diagnosis?
(A) Adjustment disorder with depressed mood
(B) Bipolar disorder
(C) Dysthymic disorder
(D) Major depressive disorder
(E) Substance abuse
3. A previously healthy 18-year-old woman is brought to the
physician for evaluation because of loss of appetite,
sleeplessness, and extreme irritability for 3 weeks. After
missing many practices, she quit the college softball team that
she previously enjoyed. She often feels tired and has
difficulty sitting still and concentrating on schoolwork. Her
menses occur at regular intervals. She weighs 50 kg (110 lb)
and is 168 cm (5 ft 6 in) tall; her BMI is 18 kg/m2. Her pulse
is 74/min, respirations are 16/min, and blood pressure is
110/70 mm Hg. Which of the following is the most likely
diagnosis?
(A) Adjustment disorder with mixed disturbance of
emotions and conduct
(B) Anorexia nervosa
(C) Attention-deficit/hyperactivity disorder
(D) Dysthymic disorder
(E) Major depressive disorder
4. A 57-year-old man comes to the physician accompanied by
his wife because of a 2-year history of fatigue. He reports
waking up tired nearly every morning, often with a headache.
He naps almost every afternoon. He thinks that the fatigue is
affecting his concentration and performance at work. His wife
says that he snores frequently during the night and sometimes
wakes up gasping for air. She describes him as a restless
sleeper. His tonsils and adenoids were removed when he was a
child. He has no history of serious illness and takes no
medications. He is 178 cm (5 ft 10 in) tall and weighs 115 kg
(253 lb); BMI is 36 kg/m2. His pulse is 86/min, and blood
pressure is 164/88 mm Hg. The nasal septum is at the midline.
Examination shows no other abnormalities. Which of the
following is the most likely diagnosis?
(A) Chronic fatigue syndrome
(B) Narcolepsy
(C) REM sleep behavior disorder
(D) Restless legs syndrome
(E) Sleep apnea
5. A 52-year-old woman whose husband died 2 months ago
consults a physician because of headaches and feelings of

uncertainty. She describes the headaches as a band around her


head; they occur unpredictably and are not accompanied by
any other symptoms. She has no history of psychiatric illness.
While talking with the physician, the patient begins to cry and
talk about her deceased husband; she feels her life is empty
now and worries about her future. Which of the following is
most appropriate at this point?
(A) Allow her to express herself
(B) Prescribe an antianxiety drug
(C) Prescribe an antidepressant drug
(D) Refer her for psychological testing
(E) Obtain a psychiatric consultation
6. A 47-year-old man is brought to the emergency department by
police after he was found eating garbage from a dumpster
behind a restaurant. He says that he just came to this town and
that he is homeless, so he has no money for food. He admits to
several psychiatric hospitalizations in the past but says that he
no longer needs medication. He appears dirty and is
malodorous. Vital signs are within normal limits. Physical
examination shows no abnormalities. On mental status
examination, his speech is clear, but his thought process is
disorganized with many loose associations. At several times
during the interview, he appears to be preoccupied with
internal stimuli. He says that he hears voices having an
ongoing conversation in his head. Which of the following is
the most likely diagnosis?
(A) Bipolar disorder
(B) Brief psychotic disorder
(C) Delusional disorder
(D) Major depressive disorder with psychotic features
(E) Psychotic disorder due to a general medical
condition
(F) Schizophrenia
(G) Schizotypal personality disorder
7. A 32-year-old woman is brought to the emergency department
because of fever, hallucinations, agitation, and confusion for 8
hours. She has a history of alcohol, cocaine, and
benzodiazepine abuse. Her temperature is 37.8C (100F),
pulse is 110/min, respirations are 16/min, and blood pressure
is 150/90 mm Hg. Examination shows tremors and
telangiectasia. The lungs are clear to auscultation. There is a
holosystolic murmur; the abdomen is tender, and the liver
edge is palpable 3 cm below the right costal margin. Rectal
examination shows no abnormalities. Her serum alkaline
phosphatase activity is 200 U/L, serum ALT activity is 60
U/L, and serum AST activity is 90 U/L. Which of the
following is the most likely cause of this condition?
(A) Acute cocaine toxicity
(B) Alcohol withdrawal
(C) Benzodiazepine withdrawal
(D) Panic disorder
(E) Schizophreniform disorder
8. A 10-year-old boy is brought to the physician because of

increasing behavior problems in school since starting 5th


grade 3 months ago. His teacher states that he is unable to sit
quietly through a classroom period and frequently disrupts the
class and interrupts other children while they are talking. His
parents report that he has always been an active child and are
concerned because he is inattentive when he runs or walks.
During examination, he fidgets with his hands and feet and is
easily distracted from completing a task. Which of the
following is the most appropriate pharmacotherapy?
(A) Amitriptyline
(B) Fluoxetine
(C) Haloperidol
(D) Imipramine
(E) Methylphenidate
9. A 27-year-old woman is brought to the emergency department
1 hour after a friend found her barely arousable in her
disorderly apartment with a nearly starving cat. The patient
appears extremely thin. Her pulse is 90/min, respirations are
6/min, and blood pressure is 90/60 mm Hg. Physical
examination shows small pupils, cracked lips, and bruises and
scratches over the upper extremities. Mental status
examination shows mild obtundation, blunted affect, and
slow, incoherent speech. Which of the following is the most
appropriate next step in management?
(A) Observation in a quiet darkened room
(B) Oral administration of chlorpromazine
(C) Intramuscular administration of naloxone
(D) Intravenous administration of haloperidol
(E) Intravenous administration of lorazepam
10. A healthy 9-year-old boy is brought to the physician by his
parents because they are concerned that he dislikes attending
school. Every morning he cries and begs to stay home. He
misses school at least 1 day weekly because his mother is
exhausted from fighting with him to attend. His teachers
report that he is quiet in class and rarely participates. He has
difficulty reading at the level of his peers and lacks
confidence. At home, he tends to stay in the same room as his
mother and will sometimes follow her around the house.
When his parents plan an evening out, he often becomes
tearful and asks many questions about when they will return.
He likes to have friends over to his house and appears to enjoy
being with them. Physical examination shows no
abnormalities. During the examination, he sits on his mother's
lap and is quiet but cooperative. He makes brief eye contact
and speaks in a low volume, becoming tearful when
questioned about being away from his mother. Which of the
following is the most likely diagnosis?
(A) Dysthymic disorder
(B) Mild mental retardation
(C) Oppositional defiant disorder
(D) Reading disorder
(E) Separation anxiety disorder
(F) Social phobia

11. A 47-year-old woman is brought to the physician by her


husband because of bizarre behavior for 1 week. Her husband
says that she makes no sense when she speaks and seems to be
seeing things. She also has had difficulty sleeping for 2
months and has gained approximately 9 kg (20 lb) during the
past 5 months. During this time, she has been moody and
easily fatigued. He also notes that the shape of her face has
become increasingly round and out of proportion with the rest
of her body despite her weight gain. She has no history of
psychiatric or medical illness. She is 160 cm (5 ft 3 in) tall
and weighs 70 kg (155 lb); BMI is 28 kg/m2. Her pulse is
98/min, respirations are 8/min, and blood pressure is 148/92
mm Hg. Physical examination shows truncal obesity and
ecchymoses over the upper and lower extremities. Neurologic
examination shows no focal findings. Mental status
examination shows pressured speech and a disorganized
thought process. There is evidence of visual and auditory
hallucinations. Urine toxicology screening is negative. Which
of the following is the most likely diagnosis?
(A) Brief psychotic disorder
(B) Major depressive disorder with psychotic features
(C) Psychotic disorder due to a general medical
condition
(D) Schizophrenia
(E) Schizotypal personality disorder
12. One day after admission to the hospital for agitation and
hallucinations, a 19-year-old man has the onset of severe
muscle stiffness that prevents him from rising out of bed. At
the time of admission, treatment with haloperidol was begun.
Today, he appears lethargic and diaphoretic. His temperature
is 39.7C (103.5F), pulse is 120/min, and blood pressure is
160/110 mm Hg. Physical examination shows generalized
severe rigidity of the upper extremities bilaterally. On mental
status examination, he is not oriented to person, place, and
time. Which of the following is the most appropriate next step
in management?
(A) Observation only
(B) Add fluoxetine
(C) Add lithium carbonate
(D) Discontinue haloperidol
(E) Increase the dosage of haloperidol
13. A 32-year-old woman comes to the physician because of a 3week history of depressed mood. She works as a local news
anchor. She says that she has always had a busy schedule, but
lately she has not had her usual amount of energy and has had
difficulty getting up and going to work. She describes herself
as normally a "hyper" person with energy to perform multiple
tasks. During the past 10 years, she has had similar episodes
in which she has had depressed mood associated with a
decreased energy level that makes her feel "slowed down."
The episodes never last more than a few weeks. She
sometimes goes through periods when she feels a surge in
energy, sleeps very little, feels at the top of her mental
powers, and is able to generate new ideas for the news station;

these episodes never last more than 5 days. She says that she
loves feeling this way and wishes the episodes would last
longer. She takes no medications. She does not drink alcohol
or use illicit drugs. Her temperature is 37C (98.6F), pulse is
70/min, and blood pressure is 125/80 mm Hg. Physical
examination shows no abnormalities. Mental status
examination shows a depressed mood and flat affect. Which
of the following is the most likely diagnosis?
(A) Attention-deficit/hyperactivity disorder
(B) Cyclothymic disorder
(C) Dysthymic disorder
(D) Major depressive disorder
(E) Mood disorder due to a general medical condition
14. A 77-year-old woman is brought to the emergency department
by her husband because of agitation and confusion for 3
hours. He states that she has been intermittently crying out
and does not appear to recognize him. A routine health
maintenance examination 3 days ago showed no abnormalities
except for mild memory deficits. Her current temperature is
37.8C (100F), pulse is 100/min, respirations are 14/min, and
blood pressure is 130/60 mm Hg. Physical examination shows
no abnormalities except for mild tenderness to palpation of the
lower abdomen. Mental status examination shows confusion;
she is oriented to person but not to time or place. Which of the
following is the most appropriate next step in diagnosis?
(A) Determination of erythrocyte sedimentation rate
(B) Measurement of serum alkaline phosphatase
activity
(C) Measurement of serum folate concentration
(D) Urinalysis
(E) Western blot assay
15. A 14-year-old boy is brought to the physician by his mother
after she found an unsmoked marijuana cigarette in his
bedroom. The mother reports that her son has never done
anything like this before. His academic performance is
excellent. When interviewed alone, the patient reports that his
friends heard about smoking marijuana and acquired some
from their peers to find out what it was like. He adds that he
has never smoked marijuana before. He requests that his
teachers not be informed because they would be very
disappointed if they found out. Physical examination shows
no abnormalities. On mental status examination, he is pleasant
and cooperative and appears remorseful. Which of the
following is the most likely diagnosis?
(A) Conduct disorder
(B) Marijuana abuse
(C) Marijuana dependence
(D) Parent-child relational problem
(E) Normal adolescence
16. An otherwise healthy 27-year-old man is referred to a
cardiologist because of three episodes of severe palpitations,
dull chest discomfort, and a choking sensation. The episodes
occur suddenly and are associated with nausea, faintness,

trembling, sweating, and tingling in the extremities; he feels


as if he is dying. Within a few hours of each episode, physical
examination and laboratory tests show no abnormalities. He
does not abuse drugs or alcohol and has no history of
interpersonal problems. Which of the following is the most
likely diagnosis?
(A) Delusional disorder
(B) Generalized anxiety disorder
(C) Hypochondriasis
(D) Panic disorder
(E) Somatization disorder
17. A 42-year-old woman is brought to the physician by her
husband because of persistent sadness, apathy, and tearfulness
for the past 2 months. She has a 10-year history of systemic
lupus erythematosus poorly controlled with corticosteroid
therapy. Physical examination shows 1-cm erythematous
lesions over the upper extremities and neck and a malar
butterfly rash. On mental status examination, she appears
depressed. She says that she would be better off dead. Which
of the following is the most appropriate next step in
management?
(A) Ask the patient about her suicidal thoughts
(B) Reassure the patient that she will get well
(C) Recommend psychiatric hospitalization
(D) Begin paroxetine therapy
(E) Increase the dose of corticosteroid
18. A 27-year-old man is brought to the emergency department by
police 2 hours after threatening his next door neighbor. The
neighbor called the police after receiving a note demanding
that she stop videotaping all of the activities in the patient's
home or he would call the police. During the examination, the
patient is cooperative. He explains that he has lived in the
neighborhood for 8 months. Three months ago, he noticed that
his neighbor installed a new satellite dish and says that since
that time, she has been watching every move he makes. He
reports no personal or family history of psychiatric illness. He
has not had changes in sleep pattern and performs well in his
job as a car salesman. He appears neatly dressed. Physical
examination shows no abnormalities. On mental status
examination, his thought process is organized and logical.
There is no evidence of suicidal or homicidal ideation or
hallucinations. He says that he is not suspicious of anyone
other than his neighbor. Which of the following is the most
likely diagnosis?
(A) Bipolar disorder
(B) Brief psychotic disorder
(C) Delusional disorder
(D) Major depressive disorder with psychotic features
(E) Schizophrenia
19. A 9-year-old girl is brought to the physician by her adoptive
parents because they are concerned about her increasing
difficulty at school since she began third grade 7 weeks ago.
Her teachers report that she is easily frustrated and has had

difficulty reading and paying attention. She also has had


increased impulsivity and more difficulty than usual making
and keeping friends. Her biologic mother abused multiple
substances before and during pregnancy, and the patient was
adopted shortly after birth. She is at the 20th percentile for
height and 40th percentile for weight. Examination shows a
flattened nasal bridge and a long philtrum. During the
examination, she is cheerful. Psychoeducational testing shows
an IQ of 82. The most likely explanation for these findings is
in utero exposure to which of the following?
(A) Alcohol
(B) Cocaine
(C) Ecstasy (3,4-methylenedioxymethamphetamine)
(D) Heroin
(E) Marijuana
(F) PCP (phencyclidine)
(G) Toluene
20. A 77-year-old man comes to the physician with his daughter
for a follow-up examination to learn the results of
neuropsychological testing performed 1 week ago for
evaluation of a recent memory loss. Results of the testing
indicated cognitive changes consistent with early stages of
dementia. Three weeks ago, he received the diagnosis of
prostate cancer and has shown signs of a depressed mood
since then. Twenty years ago, he required treatment in a
hospital for major depressive disorder. His symptoms resolved
with antidepressant therapy, and he has not taken any
psychotropic medication for the past 15 years. The patient's
daughter comes into the examination room before her father
and asks that the physician not tell her father any information
that might be upsetting, given his vulnerability to depression.
She says she is concerned about what the results might be and
how her father will handle them. The patient enters the room
soon after his daughter makes her request. Which of the
following is the most appropriate initial physician statement to
this patient?
(A) "Because of your history of depression, I would
like you to start on an antidepressant medication
before we talk any further about your
neuropsychological testing."
(B) "Before going over your test results, I'd like to
hear how you have been doing. You have been
through a difficult time."
(C) "I would like to talk with your daughter a bit about
your test results, and then I will go over things
with you."
(D) "Your daughter is concerned about you. I think
you need to see a psychiatrist before we go any
further here."
(E) "Your tests were inconclusive, and I would like to
have you start on a medication to help with your
memory as a precaution."

1. A previously healthy 17-year-old girl is brought to the physician for evaluation because of loss of appetite,
sleeplessness, and extreme irritability for 3 weeks. After missing many practices, she quit the softball team that she
previously enjoyed. She often feels tired and has difficulty sitting still and concentrating on schoolwork. Her menses
occur at regular intervals. She weighs 50 kg (110 lb) and is 168 cm (66 in) tall. Her blood pressure is 110/70 mm
Hg, pulse is 74/min, and respirations are 16/min. Which of the following is the most likely diagnosis?
(A) Adjustment disorder with mixed disturbance of emotions and conduct
(B) Anorexia nervosa
(C) Attention-deficit/hyperactivity disorder
(D) Dysthymic disorder
(E) Major depressive disorder
2. A 45-year-old man is brought to the physician by his spouse. He has been drinking heavily since he was passed
over for a job promotion 3 days ago. He stayed in bed over the weekend. He has no personal history of psychiatric
disorders and no personal or family history of alcohol abuse. He is crying and states, "I can't believe it," when
addressed. When asked what he will do, he states, "I don't know, but if I don't go back to work tomorrow, I'll lose my
job." Which of the following is the most likely diagnosis?
(A) Adjustment disorder with depressed mood
(B) Bipolar disorder
(C) Dysthymic disorder
(D) Major depressive disorder
(E) Substance abuse
3. A 52-year-old woman whose husband died 2 months ago consults a physician because of headaches and feelings
of uncertainty. She describes the headaches as a band around her head; they occur unpredictably and are not
accompanied by any other symptoms. She has no history of psychiatric illness. While talking with the physician, the
patient begins to cry and talk about her deceased husband; she feels her life is empty now and worries about her
future. Which of the following is most appropriate at this point?
(A) Allow her to express herself
(B) Prescribe an antianxiety drug
(C) Prescribe an antidepressant drug
(D) Refer her for psychological testing
(E) Obtain a psychiatric consultation
4. A 10-year-old boy is brought to the physician because of increasing behavior problems in school since starting 5th
grade 3 months ago. His teacher states that he is unable to sit quietly through a classroom period and frequently
disrupts the class and interrupts other children while they are talking. His parents report that he has always been an
active child and are concerned because he is inattentive when he runs or walks. During the examination, he fidgets
with his hands and feet and is easily distracted from completing a task. Which of the following is the most
appropriate pharmacotherapy?
(A) Amitriptyline
(B) Fluoxetine
(C) Haloperidol
(D) Imipramine
(E) Methylphenidate
5. A 32-year-old woman is brought to the emergency department because of fever, hallucinations, agitation, and
confusion for 8 hours. She has a history of alcohol, cocaine, and benzodiazepine abuse. Her temperature is 37.8 C
(100 F), blood pressure is 150/90 mm Hg, pulse is 110/min, and respirations are 16/min. She is tremulous. The lungs
are clear to auscultation. She has a holosystolic murmur; the abdomen is tender, and the liver edge is palpable 3 cm
below the costal margin. Rectal examination shows no abnormalities. She has telangiectasia. A complete blood
count and liver function tests show no abnormalities. Her serum alkaline phosphatase activity is 200 U/L, serum
alanine aminotransferase (ALT, GPT) activity is 60 U/L, and serum aspartate aminotransferase (AST, GOT) activity
is 90 U/L. Which of the following is the most likely cause of this condition?
(A) Acute cocaine toxicity
(B) Alcohol withdrawal
(C) Benzodiazepine withdrawal

(D) Panic disorder


(E) Schizophreniform disorder
Answers to first set of questions:
A1) c - Reactive attachment disorder results from a severely dysfunctional early relationship between the principal
caregiver and the child. When caregivers consistently disregard the child's physical or emotional needs, the child
fails to develop a secure and stable attachment with them. This failure causes a severe disturbance of the child's
ability to relate to others, manifested in a variety of behavioral and interpersonal problems. Some children are
fearful, inhibited, withdrawn, and apathetic; others are aggressive, disruptive, and disorganized with low frustration
tolerance and poor affect modulation. This condition is often confused with ODD or ADHD.
A2) d - Treating new onset feelings of depression and anhidonia with an SSRI. he goal of secondary prevention is to
address problems at an early stage to avoid future more severe problems.
A3) e- Circumstantiality is a disturbance in which the patient digresses into unnecessary details before
communicating the central idea.
Answer Key for Psychiatry Sample Questions
(Questions 1-20)
1. A
2. A
3. E
4. E
5. A
6. F
7. B
8. E
9. C
10. E
11. C
12. D
13. B
14. D
15. E
16. D
17. A
18. C
19. A
20. B
Last 5 sample questions
1.
2.
3.
4.
5.

E
A
A
E
B

You might also like